a我考网

 找回密码
 立即注册

QQ登录

只需一步,快速开始

扫一扫,访问微社区

查看: 314|回复: 0

法学院入学考试全真试题测试(二)b

[复制链接]
发表于 2012-8-15 13:26:27 | 显示全部楼层 |阅读模式
Question 13-18 3 T& X: |+ w6 y
+ |& x$ T. D( D. _( p! V
On an undeveloped street, a developer will simultaneously build four houses on one side, numbered consecutively 1, 3, 5, and 7, and four houses on the opposite side, numbered consecutively 2, 4, 6, and 8. Houses 2, 4, 6, and 8 will face houses 1, 3, 5, and 7, respectively. Each house will be exactly one of three styles-ranch, split-level or Tudor-according to the following conditions: 4 A  S* ~' q. b
  w+ y$ b2 i8 C0 l( S
  Adjacent houses are of different styles.
6 o: m0 `8 C& b6 [& I! f/ R, a5 U( L) f9 M9 _7 u
  No split-level house faces another split-level house. ) z( J: J3 H4 v& S
 
" V; H9 @6 u  D: t  Every ranch house has at least one Tudor house adjacent to it 3 A' |4 }) t4 \# i
3 d4 r1 x% G1 H# o$ D3 n$ y
  House 3 is a ranch house. " J$ m+ V9 U' u& w% h0 |
6 i( Y5 M% c. B0 i# P: R" k# e3 f
  House 6 is a split-level house.
' `: x, Z2 M' s+ o+ n, |: k. c2 n$ C1 p* J  t- P$ R. n  `
13. Any of the following could be a Tudor house EXCEPT house
- D! D5 }! _. ]3 q$ F; g! A1 ^. B6 f+ @, |) a# X3 N$ K- G% h
  (A) 1
4 t' L  u) m3 O! C7 t) D  (B) 2 / e2 k6 L1 n' l! L* S
  (C) 4 ' _# G0 G. ?$ w" a2 q/ ]) L% \! G
  (D) 7
. j% h5 M9 z. E, T1 A: x, `  (E) 8
; n: S1 i9 Z6 O2 {- V4 @8 y
3 ], L8 M3 _1 j; H. F- \14. If there is one ranch house directly opposite another ranch house, which   one of the following could be true? ! w, w% U; l$ D
/ C9 V0 b8 e7 o+ k' n
  (A) House 8 is a ranch house
  ?" ^3 x; K( x. Q* Z' B% p0 y  (B) House 7 is a split-level house.
6 U/ @1 s. G+ A6 r& W  (C) House 4 is a Tudor house.
5 E$ i4 Z0 P" O( ^* B6 b- q8 w  (D) House 2 is a split-level house.
1 ~) ?  U  s3 ^. c  C  (E) House 1 is a ranch house. % Q1 R) l0 g: ?6 ]- U& O  K
, N0 _# f  X, k" a2 X
15. If house 4 is a Tudor house, then it could be true that house 0 }& s) v4 F" m+ @( P' h$ ?
3 r4 _. E! n$ }6 C3 [8 F
  (A) 1 is a Tudor house  
5 u' V2 a) u# x8 o% I9 R' u! ^  (B) 2 is a Tudor house  
# o1 z7 m, \5 j  (C) 5 is a Tudor house  2 m3 D8 ?) E: l% \4 _9 a1 j' u
  (D) 7 is a Tudor house  1 ^/ V. |& g: n
  (E) 8 is a Tudor house  % r" j4 s- V; c( O1 a& L: K% q

/ x; ?  H: H; _+ z4 {* n$ y16. On the street, there could be exactly.
. o) Q9 `, H9 u$ H5 y1 _* I6 _/ _' H
  (A) one ranch house % m! ?3 x) r. W, G! W1 {
  (B) one Tudor house $ c0 W, L6 I- [; \+ t
  (C) two Tudor houses
9 P2 ?; }2 U. ^! z6 T- j# T; @3 {) Z* J  (D) four ranch houses  & C( f7 o3 h5 C& J& [2 a5 Z# p
  (E) five ranch houses 1 p5 k  p0 `9 x; M5 m; @& w- X

0 \- p9 n3 J$ y  L% Q* j0 O* z$ v17. If no house faces a house of the same style, then it must be true that  
, U' j, x0 s8 j9 ^3 _  house
+ H0 r5 z9 f" ^0 f0 @5 s9 \& ~3 p( \9 t( \* B; \
  (A) 1 is a split-level house 6 A- z( p2 `7 _7 B2 U" ~8 m
  (B) 1 is a Tudor house
. C) q4 A. x9 y/ n& a' w7 \, I" I  (C) 2 is a ranch house  2 D7 z) l+ W" j$ h: X7 m  s
  (D) 2 is a split-level house . e7 U+ g( W( p
  (E) 4 is a Tudor house
9 O, W! z+ C% ?4 S: q# W8 c4 T, ^" Q: u8 b
18. If the condition requiring house 6 to be a split-level house is suspended but all other original conditions remain the same, the any of the following could be an accurate list of the styles of houses 2, 4, 6, and 8, respectively, EXCEPT:
. O, l0 F& m# Q# V9 Q$ {  ' N1 U  m8 c# s4 i" b/ ?. n
  (A) ranch, split-level, ranch, Tudor
) N. H! g, o0 a. g/ C  (B) split-level, ranch, Tudor, split-level
/ Z8 M0 G0 [, \0 T1 f  (C) split-level, Tudor, ranch, split-level & u8 r% O( A* q; u) ^* D
  (D) Tudor, ranch. Tudor, split-level
2 B" a6 P6 ]* q- x  (E) Tudor split-level, ranch, Tudor
* ~: h* d" ^/ s
' c$ H- M- [0 p: O, O/ F3 P  WQuestions 19-24 1 b# j$ ]& b- ?! q) `+ S( G
' H+ m; c' L* `3 m; x( Z; F
Within a tennis league each of five teams occupies one of five positions, numbered 1 through 5 in order of rank, with number 1 as the highest position. The teams are initially in the order R, J, S, M, L, with R in position 1. Teams change positions only when a lower-positioned team defeats a higher-positioned team. The rules are as follows: 0 ?1 q, e) }1 c5 D

' j: U8 Y2 `; u4 p' l) M  Matches are played alternately in odd-position rounds and in even-position rounds.
: _; c3 c3 o( z0 b
# J1 X, r7 p. t% ]5 Z7 i  In an odd-position round, teams in position 3 and 5 play against teams positioned immediately above them. % O! D6 z$ q8 w* S; b

  E# k6 A9 Z2 A, D. P  In an even-position round, teams in position 2 and 4 play against teams positioned immediately above them.   l5 o8 g6 z/ s6 x; P
7 [; H8 _9 o5 v5 Q* o1 q5 j/ j
  When a lower-positioned team defeats a higher-positioned team, the two teams switch positions after the round is completed.
$ d" y3 h$ T3 J* }- |
( N; M  k4 M& ]: f, v19. Which one of the of following could be the order of teams, from position 1 through position 5 respectively, after exactly one round of even-position matches if no odd-position round has yet been played? % W& z* e2 M9 x) I) H  S
2 i( y1 n& ]  K$ }" W6 |
  (A) J , R, M, L, S
) ]; G) f' p* T( N  (B) J, R, S, L, M
& b$ |- y7 c% U1 L, H' o- ]) f  (C) R, J, M, L, S ( [. v; u! l. e4 t4 P
  (D) R, J, M, S, L  8 |, \0 v7 A6 @$ ^) E9 j
  (E) R, S, J, L, M
) k; `% p7 D3 Y9 y; D( S! A- V. m  ~' g
20. If exactly two rounds of matches have been played, beginning with an odd-position round, and if the lower-positioned teams have won every match in those two rounds, then each of the following must be true EXCEPT: $ s5 j0 k1 }) N1 Z$ o) f

  _1 Y0 F8 X5 p( n* e9 e  (A) L is one position higher than J.
" z, Y$ _# ~6 d$ x, V  (B) R is one position higher than L.
; i6 e( t' Z4 a+ n4 `  g& I  (C) S is one position higher than R. ) R/ q! v  d! _8 u# P5 Q
  (D) J is in position 4. 3 x1 p! _4 r! U0 l7 Y2 Q% X  W: r$ u1 F
  (E) M is position 3. * B9 U+ x- Z  {+ g6 y1 S4 X: p

: e% m# g9 j* F21. Which one of the following could be true after exactly two rounds of matches have been played?
* g' L  a% D) R, I. n
1 J7 h6 z  y+ |$ s  {' D7 x& m- p  (A) J has won two matches.
- @1 m$ M% j; D* p( W) W, C% D  (B) L has lost two matches. : e5 N$ {) h: |( q
  (C) R has won two matches.
3 w1 ]  a( P. r% B3 G% o5 |! K  (D) L’s only match was played against J. 6 R1 U2 ?, L& o& ^, l$ v* E4 |+ W
  (E) M played against S in two matches. $ @- z& ?/ m8 n, U* n

- _/ H4 X4 W. R22. If after exactly three rounds of matches M is in position 4, and J and L have won all of their matches, then which one of the following can be true?
6 _: v, N3 W/ {+ |0 j" l$ b
4 y' F$ l5 ^- u' u- L, p  (A) J is in position 2. 6 `1 V6 F+ z+ |! X  L' Y' l
  (B) J is in position 3.
( y% i( P( A4 Y+ _  (C) L is in position 2.
* ^- [5 L8 O) N  (D) R is in position 1. 2 n' j4 D. @$ v' Y" N$ m+ m. N
  (E) S is in position 3.
1 p2 @7 L. i' s2 U' P1 ^9 }$ n) S
/ H2 E" K; E0 e23. If after exactly three rounds M has won three matches and the rankings of the other four teams relative to each other remain the same, then which one of the following must be in position 3? " l5 {4 G2 @) p/ {* N' L3 `
  4 v$ P/ J2 R* B2 w; G
  (A) J / i5 E# |5 e& H, p' M' C1 {
  (B) L
4 e0 [8 J, j4 q" q* p  (C) M & @6 t$ M$ m/ A0 H
  (D) R
& r: z* i+ N; p$ s. V7 F) N  (E) S
# q6 u+ X( o0 ^* s" n) B* @$ E& U9 |) W# |
24. If after exactly three rounds the teams, in order from first to fifth position, are R, J, L, S, and M, then which one of the following could be the order, from first to fifth position, of the teams after the second round?
. b4 V' G' W: M) R3 M' i
. X: z. _! P& X+ J! N" {% r  (A) J, R, M, S, L
! B  G7 ]  X9 z2 |. Q- B2 r- n  (B) J, L, S, M, R 7 ~! v2 u7 y+ E! ]# V/ |0 b
  (C) R, J, S, L, M 7 ]/ `6 G: K5 x% Z6 Y5 j
  (D) R, L, M, S, J & B6 ~+ H* U$ s9 N
  (E) R, M, L, S, J
回复

使用道具 举报

您需要登录后才可以回帖 登录 | 立即注册

本版积分规则

Archiver|手机版|小黑屋|Woexam.Com ( 湘ICP备18023104号 )

GMT+8, 2024-4-25 09:17 , Processed in 0.304187 second(s), 21 queries .

Powered by Discuz! X3.4 Licensed

© 2001-2017 Comsenz Inc.

快速回复 返回顶部 返回列表